完整版真题免费下载
+答案解析请参考文末
Are there integers and
such that
and
are both perfect cubes of integers?
Each cell of an board is filled with some nonnegative integer. Two numbers in the filling are said to be adjacent if their cells share a common side. (Note that two numbers in cells that share only a corner are not adjacent). The filling is called a garden if it satisfies the following two conditions:
(i) The difference between any two adjacent numbers is either or
.
(ii) If a number is less than or equal to all of its adjacent numbers, then it is equal to .
Determine the number of distinct gardens in terms of and
.
In triangle , points
lie on sides
respectively. Let
,
,
denote the circumcircles of triangles
,
,
, respectively. Given the fact that segment
intersects
,
,
again at
respectively, prove that
.
Let be the number of ways to write
as a sum of powers of
, where we keep track of the order of the summation. For example,
because
can be written as
,
,
,
,
, and
. Find the smallest
greater than
for which
is odd.
Quadrilateral is inscribed in the semicircle
with diameter
. Segments
and
meet at
. Point
is the foot of the perpendicular from
to line
. Point
lies on
such that line
is perpendicular to line
. Let
be the intersection of segments
and
. Prove that
、
Find all real numbers satisfying
No, such integers do not exist. This shall be proven by contradiction, by showing that if is a perfect cube then
cannot be.
Remark that perfect cubes are always congruent to ,
, or
modulo
. Therefore, if
, then
.
If , then note that
. (This is because if
then
.) Therefore
and
, contradiction.
Otherwise, either or
. Note that since
is a perfect sixth power, and since neither
nor
contains a factor of
,
. If
, then
Similarly, if
, then
Therefore
, contradiction.
Therefore no such integers exist.
We shall prove that such integers do not exist via contradiction. Suppose that and
for integers x and y. Rearranging terms gives
and
. Solving for a and b (by first multiplying the equations together and taking the sixth root) gives a =
and b =
. Consider a prime p in the prime factorization of
and
. If it has power
in
and power
in
, then
-
is a multiple of 24 and
-
also is a multiple of 24.
Adding and subtracting the divisions gives that -
divides 12. (actually,
is a multiple of 4, as you can verify if
. So the rest of the proof is invalid.) Because
-
also divides 12,
divides 12 and thus
divides 3. Repeating this trick for all primes in
, we see that
is a perfect cube, say
. Then
and
, so that
and
. Clearly, this system of equations has no integer solutions for
or
, a contradiction, hence completing the proof.
Therefore no such integers exist.
Let and
. Then,
,
, and
Now take
(recall that perfect cubes
and perfect sixth powers
) on both sides. There are
cases to consider on what values
that
and
take. Checking these
cases, we see that only
or
yield a valid residue
(specifically,
). But this means that
, so
so
contradiction.
If is a perfect cube, then
can be one of
, so
can be one of
,
, or
. If
were divisible by
, we'd have
, which we've ruled out. So
, which means
, and therefore
.
We've shown that can be one of
, so
can be one of
. None of these are possibilities for a perfect cube, so if
is a perfect cube,
cannot be.
As in previous solutions, notice . Now multiplying gives
, which is only
, so after testing all cases we find that
. Then since
,
and
(Note that
cannot be
). Thus we find that the inverse of
is itself under modulo
, a contradiction.
We claim that any configuration of 's produces a distinct garden. To verify this claim, we show that, for any cell that is nonzero, the value of that cell is its distance away from the nearest zero, where distance means the shortest chain of adjacent cells connecting two cells. Now, since we know that any cell with a nonzero value must have a cell adjacent to it that is less than its value, there is a path that goes from this cell to the
that is decreasing, which means that the value of the cell must be its distance from the
as the path must end. From this, we realize that, for any configuration of
's, the value of each of the cells is simply its distance from the nearest
, and therefore one garden is produced for every configuration of
's.
However, we also note that there must be at least one in the garden, as otherwise the smallest number in the garden, which is less than or equal to all of its neighbors, is
, which violates condition
. There are
possible configurations of
and not
in the garden, one of which has no
's, so our total amount of configurations is
Note: "bordering" and "surrounding" mean that two cells have to touch on a side; one vertex is not good enough.
We note that there is no real step to begin the problem, so start by constructing the base case: put ALL the zeroes into the board. Then note that all squares bordering it (corners alone don't count) have to be . After doing that, what can a cell bordering a 1 have as its value? Either 1 or 2, based on (i). But if it were 1, then all cells surrounding it would have at least 1 as their value by (i). And by (ii) the value would have to be 0, which contradicts our initial construction. Therefore, all the cells bordering a 1 have to be 2. Looks too simple for JMO, right? We apply the same logic to a cell bordering value
: either it is
or
. If it is
, however, by constraint (i) and (ii) we realize that this cell has to be 0 (because neighbors cannot be less than it), contradicting our construction again! Therefore, that cell has to be
. And so on until the grid is filled. Basically the problem reduces to finding all 0s, surrounding them with 1s, and surrounding the 1s with 2s, etc. Beautiful! I have established a bijection between the zeros placed in the grid and the arrangement, therefore there are
solutions right? NO! Take the smallest cell. It has to be
via criterion (ii). So a case with zero zeros is apocryphal. Our answer is
.
In this solution, all lengths and angles are directed.
Firstly, it is easy to see by that concur at a point
. Let
meet
again at
and
, respectively. Then by Power of a Point, we have
Thusly
But we claim that
. Indeed,
and
Therefore,
. Analogously we find that
and we are done.
Diagram Refer to the Diagram link.
By Miquel's Theorem, there exists a point at which intersect. We denote this point by
Now, we angle chase:
In addition, we have
Now, by the Ratio Lemma, we have
(by the Law of Sines in
)
(by the Law of Sines in
)
by the Ratio Lemma. The proof is complete.
Use directed angles modulo .
Lemma.
Proof.
Now, it follows that (now not using directed angles)using the facts that
and
,
and
are similar triangles, and that
equals twice the circumradius of the circumcircle of
.
First of all, note that =
where
is the largest integer such that
. We let
for convenience.
From here, we proceed by induction, with our claim being that the only such that
is odd are
representable of the form
We induct on . It is trivially true for
and
. From here, we show that, if the only numbers
where
is odd are of the form described above, then the only numbers
that are odd are of that form. We first consider all numbers
, such that
, going from the lower bound to the upper bound (a mini induction, you might say). We know that
. For a number in this summation to be odd,
. However, we know that
, so
must be equal to
, or else
cannot be in that interval. Now, from this, we know that
, as
. Therefore,
and
are distinct, and thus
and
are odd; since there are just two odd numbers, the ending sum for any
is even. Finally, considering
, the only odd number is
, so the ending sum is odd.
The smallest greater than
expressible as
is
Of course, as with any number theory problem, use actual numbers to start, not variables! By plotting out the first few sums (do it!) and looking for patterns, we observe that , where
represents the largest power of
that is smaller than
. I will call this sum the Divine Sign, or DS.
But wait a minute... we are trying to determine odd/even of . Why not call all the evens 0 and odds 1, basically using mod 2? Sounds so simple. Draw a small table for the values: as
goes up from
, you get:
. We have to set
for this to work. Already it looks like
is only odd if
.
The only tool here is induction. The base case is clearly established. Then let's assume we successfully made our claim up to . We need to visit numbers from
to
. Realize that
has
for
because there will be two numbers in DS that give a
of one:
and
.
But to look at whether a value of is 1 or 0, we need to revisit our first equation. We can answer this rather natural question: When will a number to be inducted upon, say
, ever have a 1 as
in the DS equation? Well- because by our assumption of the claim up to
, we know that the only way for that to happen is if
in the DS is equal to
. Clearly
.
Finally, we can simplify. Using our last equation, , regrouping gives
.
Most importantly, realize that can be from
to
, because of the restraints on
mentioned earlier. Same with
. Immediately at least one of
and
has to be
. If both were smaller, LHS is greater, contradiction. If both were greater, RHS is greater, contradiction.
Therefore, by setting one of or
to
, we realize
.
The conclusion is clear, right? Each from
to
yields two distinct cases: one of
and
is equal to
, while the other is LESS THAN
. But for
, there is ONE CASE: BOTH values have to equal
. Therefore, the only
that has
as odd must only be
, because the other ones yield a
of 1+1=0 in our mod. That proves our induction for a new power of 2, namely
, meaning that
is only odd if
, and we are almost done...
Thus, the answer is .
This was pretty intuitive, and realizing quite steps about how powers of 2 work gave us the solution! Estimated time: ~50 minutes. Cheers- expiLnCalc.
Let us use coordinates. Let O, the center of the circle, be (0,0). WLOG the radius of the circle is 1, so set Y (1,0) and X (-1,0). Also, for arbitrary constants and
set A
and B
. Now, let's use our coordinate tools. It is easily derived that the equation of
is
and the equation of
is
, where
and
are defined appropriately. Thus, by equating the y's in the equation we find the intersection of these lines,
, is
. Also,
. It shall be left to the reader to find the slope of
, the coordinates of Q and C, and use the distance formula to verify that
.
First of all
since the quadrilateral
is cyclic, and triangle
is rectangle, and
is orthogonal to
. Now
because
is cyclic and we have proved that
so
is parallel to
, and
Now by Ptolomey's theorem on
we have
we see that triangles
and
are similar since
and
is already proven, so
Substituting yields
dividing by
We get
Now triangles
, and
are similar so
but also triangles
and
are similar and we get
Comparing we have,
Substituting,
Dividing the new relation by
and multiplying by
we get
but
since triangles
and
are similar, because
and
since
Substituting again we get
Now since triangles
and
are similar we have
and by the similarity of
and
, we get
so substituting, and separating terms we get
In the beginning we prove that
and
so
It is obvious thatfor some value
. Also, note that
. Set
We have
and
This gives
Similarly, we can deduce that
Adding gives
First, since is the diameter and
,
, and
lie on the circle,
. Next, because
and
are both perpendicular to
, we have
to be parallel to
.
Now looking at quadrilateral , we see that this is cyclic because
Set
, and
. Now,
since
and
are parallel. Also,
That means
so
This means
, so
and
are parallel. Finally, we can look at the equation. We know
so
We also know
so
Plugging this into the LHS of the equation, we get
Now, let
be the point on
such that
is perpendicular to
. Also, since
, their arcs have equal length, and
. Now, the LHS is simplified even more to
which is equal to
which is equal to
This completes the proof.
The key Lemma is:for all
. Equality holds when
.
This is proven easily.by Cauchy. Equality then holds when
.
Now assume that . Now note that, by the Lemma,
. So equality must hold. So
and
. If we let
, then we can easily compute that
. Now it remains to check that
.
But by easy computations, , which is obvious. Also
, which is obvious, since
.
So all solutions are of the form , and all permutations for
.
Remark: An alternative proof of the key Lemma is the following: By AM-GM,. Now taking the square root of both sides gives the desired. Equality holds when
.
Without loss of generality, let . Then
.
Suppose x = y = z. Then , so
. It is easily verified that
has no solution in positive numbers greater than 1. Thus,
for x = y = z. We suspect if the inequality always holds.
Let x = 1. Then we have , which simplifies to
and hence
Let us try a few examples: if y = z = 2, we have
; if y = z, we have
, which reduces to
. The discriminant (16 - 20) is negative, so in fact the inequality is strict. Now notice that yz - y - z + 3 = (y-1)(z-1) + 2. Now we see we can let
! Thus,
and the claim holds for x = 1.
If x > 1, we see the will provide a huge obstacle when squaring. But, using the identity
:
which leads to
Again, we experiment. If x = 2, y = 3, and z = 3, then
.
Now, we see the finish: setting gives
. We can solve a quadratic in u! Because this problem is a #6, the crown jewel of USAJMO problems, we do not hesitate in computing the messy computations:
Because the coefficient of is positive, all we need to do is to verify that the discriminant is nonpositive:
Let us try a few examples. If y = z, then the discriminant D = .
We are almost done, but we need to find the correct argument. (How frustrating!) Success! The discriminant is negative. Thus, we can replace our claim with a strict one, and there are no real solutions to the original equation in the hypothesis.
--Thinking Process by suli
WLOG, assume that . Let
and
. Then
,
and
. The equation becomes
Rearranging the terms, we have
Therefore
and
Express
and
in terms of
, we have
and
Easy to check that
is the smallest among
,
and
Then
,
and
Let
, we have the solutions for
as follows:
and permutations for all
翰林课程体验,退费流程快速投诉邮箱: yuxi@linstitute.net 沪ICP备2023009024号-1